The angles in a triangle are represented by x, x+10, and x+50. What is the measure of the largest angle?
A.70 degrees
B.80 degrees
C.100 degrees
D.90 degrees

Answers

Answer 1

Given that,

The angles in a triangle are represented by x, x+10, and x+50.

We had to,

find the measure of the largest angle.

Let's start to solve,

→ x + (x+10) + (x+50) = 180°

→ x + x + x = 180° (-50 -10)

→ 3x = 180° -60

→ 3x = 120

→ x = 120/3

→ x = 40°

Then the value of x + 10,

→ x + 10

→ 40 + 10

→ 50°

Then the value of x + 50,

→ x + 50

→ 40 + 50

→ 90°

The measure of the largest angle is,

→ D. 90 degrees

Hence, option (D) is correct answer.

Answer 2

Step-by-step explanation:

good of you and good workings


Related Questions

> There are 14 books on a shelf. 6 of these books are new. The rest of them are used (a) What is the ratio of new books to used books? (b) What is the ratio of used books to all books on the shelf ​

Answers

Answer:

a) 6:8

Because you have 14 books total if you substract 14 - 6= 8, so now you have

14 Books total

6 New Books

8 Used Books.

So, the ratio of new books to used books is 6:8 or if you simplified is 3:4.

b) 8:14

Because you have 8 used books compare to 14 books total. If you simplified your fraction you'll have 4:7

Step-by-step explanation:

Complete the table for the given rule.
Rule: y is 0.75 greater than x
x y
0
3
9

Answers

The complete table is

x    y

0    0.75

3     3.75

9     9.75

What is equation?

An equation is a mathematical statement that is made up of two expressions connected by an equal sign.

What is substitution?

Substitution means putting numbers in place of letters to calculate the value of an expression or equation.

According to the given question.

We have values of x.

Also, one rule that y is 0.75 greater than x.

So, we have a equation for finding the value of y  i.e.

[tex]y = x + 0.75..(i)[/tex]

For finding the value of y

At x = 0, substitute x = 0 in equation (i)

[tex]y = 0 + 0.75\\\implies y = 0.75[/tex]

At x = 3, substitute x = 3 in equation (i)

[tex]y = 3+0.75\\\implies y = 3.75[/tex]

At x = 9, substitute x = 9 in equation (i)

[tex]y = 9+0.75\\\implies y = 9.75[/tex]

Hence, the complete table is

x    y

0    0.75

3     3.75

9     9.75

Find out more information about equation and substitution here:

https://brainly.com/question/10852714

#SPJ2

Write the additive inverse of each of the following rational numbers:

Answers

Answer:

(1)-3/4, (2) 5/21, (3)4/43

HELPPPPP ASP PLZZZZZ

Answers

Answer:

[tex](f-g)(x)[/tex]

[tex]f(x)-g(x)[/tex]

[tex]x^{2} -6x-27-x+9[/tex]

[tex]x^{2} -7x-18[/tex]

----------------------

[tex](f*g)(x)[/tex]

[tex]=f(x)g(x)[/tex]

[tex](x^{2} -6x-27)(x-9)[/tex]

[tex]=x^{3} -15x^{2}+27x+243[/tex]

----------------------

[tex]\frac{f}{g} (x)[/tex]

[tex]\frac{x^{2} -6x-27}{x-9}[/tex]

[tex]\frac{(x-9)(x+3)}{x-9}[/tex]

[tex]x+3[/tex]

-----------------------

[tex](f+g)(x)[/tex]

[tex]f(x)+g(x)[/tex]

[tex]=x^{2} -6x-27+x-9[/tex]

[tex]=x^{2} -5x-36[/tex]

------------------------

OAmalOHopeO

------------------------

Im new to this app!
And im looking for help!!
Please help ASAP!!!
Please!!!!

Answers

y=x²-10x-7

a>0 so we will be looking for minimum

x=-b/2a=10/2=5

y=25-50-7=-32

Answer: (5;32)

Find the equation of the linear function represented by the table below in slope-intercept form.

Answers

Answer:

y=-4x-5

Step-by-step explanation:

The slope of the line is - 4, the equation of line is y=-4x-5

John runs a computer software store. Yesterday he counted 140 people who walked by the store, 63 of whom came into the store. Of the 63, only 25 bought something in the store.
(a) Estimate the probability that a person who walks by the store will enter the store. (Round your answer to two decimal places.)
(b) Estimate the probability that a person who walks into the store will buy something. (Round your answer to two decimal places.)
(c) Estimate the probability that a person who walks by the store will come in and buy something. (Round your answer to two decimal places.)
(d) Estimate the probability that a person who comes into the store will buy nothing. (Round your answer to two decimal places.)

Answers

Answer:

.................

Step-by-step explanation:

............

a. 45%
b. 39.68%
c. 17.86%
d. 60.32%

look at the image for the question

Answers

It’s 16
You would multiply height (4) by the base (length(2) x width(2)=4) which is 16

1. A helicopter is at a position from two VORS (VHF Omnidirectional
Radio Range, an aircraft navigation system operating in the VHF band -
not covered in chapter) as in the diagram shown below. Given the angles
shown, find the third angle.
Helicopter
74.0°
66.0°
VOR
VOR

Answers

The position of the helicopter and the two VORs forms a triangle and the third angle formed by these three entities is 40 degrees

The diagram is not shown; however, the question can still be answered.

The given angles are:

[tex]\theta_1 = 74.0^o[/tex]

[tex]\theta_2 = 66.0^o[/tex]

Represent the third angle as [tex]\theta_3[/tex]

The helicopter and the 2 VORs form a triangle.

So, we make use of the following theorem to calculate the third angle

[tex]\theta_1 + \theta_2 + \theta_3= 180^o[/tex] ---- sum of angles in a triangle

Substitute known values

[tex]74.0^o + 66.0^o + \theta_3= 180^o[/tex]

[tex]140.0^o + \theta_3= 180^o[/tex]

Collect like terms

[tex]\theta_3= 180 -140.0^o[/tex]

[tex]\theta_3= 40^o[/tex]

Hence, the third angle is 40 degrees.

Learn more about angles in a triangle at:

https://brainly.com/question/14780489

Find the area of the figure. (Sides meet at right angles.)

Answers

Answer:

56

Step-by-step explanation:

A=(3*4)+(4*(4+3+4))=56

Find an upper bound for E(h) the error of the machine approximation of the two-point forward difference formula for the first derivative and then find the h corresponding to the minimum of E(h).

The two-point forward difference formula for f'(x) is:_________

Answers

Answer:

I doubt it is not going to be a great

Please Help!! Whoever helps and gets it correct gets Brainliest and 5 star rating!!

Answers

Answer:

the reasoning states that "all the numbers begin with a 7 or an 8"

however this is not accurate as they can be in different placements

which can make a big difference in the total estimate.

for example:

the number could've been an 8, or an 80

they both begin with an 8

however have totally different values and could have messed up the total estimated number.

hope this helps :D

Seven and one-half foot-pounds of work is required to compress a spring 2 inches from its natural length. Find the work required to compress the spring an additional 3 inch.

Answers

Answer:

Apply Hooke's Law to the integral application for work: W = int_a^b F dx , we get:

W = int_a^b kx dx

W = k * int_a^b x dx

Apply Power rule for integration: int x^n(dx) = x^(n+1)/(n+1)

W = k * x^(1+1)/(1+1)|_a^b

W = k * x^2/2|_a^b

 

From the given work: seven and one-half foot-pounds (7.5 ft-lbs) , note that the units has "ft" instead of inches.   To be consistent, apply the conversion factor: 12 inches = 1 foot then:

 

2 inches = 1/6 ft

 

1/2 or 0.5 inches =1/24 ft

To solve for k, we consider the initial condition of applying 7.5 ft-lbs to compress a spring  2 inches or 1/6 ft from its natural length. Compressing 1/6 ft of it natural length implies the boundary values: a=0 to b=1/6 ft.

Applying  W = k * x^2/2|_a^b , we get:

7.5= k * x^2/2|_0^(1/6)

Apply definite integral formula: F(x)|_a^b = F(b)-F(a) .

7.5 =k [(1/6)^2/2-(0)^2/2]

7.5 = k * [(1/36)/2 -0]

7.5= k *[1/72]

 

k =7.5*72

k =540

 

To solve for the work needed to compress the spring with additional 1/24 ft, we  plug-in: k =540 , a=1/6 , and b = 5/24 on W = k * x^2/2|_a^b .

Note that compressing "additional one-half inches" from its 2 inches compression is the same as to  compress a spring 2.5 inches or 5/24 ft from its natural length.

W= 540 * x^2/2|_((1/6))^((5/24))

W = 540 [ (5/24)^2/2-(1/6)^2/2 ]

W =540 [25/1152- 1/72 ]

W =540[1/128]

W=135/32 or 4.21875 ft-lbs

Step-by-step explanation:

Round 573.073 to the greatest place

Answers

Answer:

574

Step-by-step explanation:

To round a two-digit number to the nearest ten, simply increase it or decrease it to the nearest number that ends in 0: When a number ends in 1, 2, 3, or 4, bring it down; in other words, keep the tens digit the same and turn the ones digit into a 0

Hope this helps <3

if U>T, R>Q, S>T and T>R, which of the following is TRUE?
1. S>Q
2. U > S
3.U > R​
A. 1 only
B. 2 only
C. 1 and 2
D. 2 and 3

Answers

Answer:

C. 1 and 2

Step-by-step explantation:

First, i would order them as U>T, T>R, R>Q, S>T

we can rewrite them as

U>T>R>Q,

now adding S, we get U>S>T>R>Q,

so U>S

We can also rewrite all of them as inequalities:

U-T>0

T-R>0

R-Q>0

S-T>0

Add R-Q and T-R

(R-Q)+(T-R)>0

-Q+T>0

T>Q, but because S>T we can say S>Q

Find 0.2B
B=[50 10
25 15]

Answers

Multiplying a matrix by a scalar results in every entry in a matrix get multiplied by that scalar, as defined,

[tex]a\begin{bmatrix}b&c\\d&e\\\end{bmatrix}=\begin{bmatrix}ab&ac\\ad&ae\\\end{bmatrix}[/tex]

So in our case, ([tex]0.2=\frac{1}{5}[/tex]

[tex]\frac{1}{5}\begin{bmatrix}50&10\\25&15\\\end{bmatrix}=\begin{bmatrix}\frac{50}{5}&\frac{10}{5}\\\frac{25}{5}&\frac{15}{5}\\\end{bmatrix}=\boxed{\begin{bmatrix}10&2\\5&3\\\end{bmatrix}}[/tex]

Hope this helps :)

Paul can install a 300-square-foot hardwood floor in 18 hours. Matt can install the same floor in 22 hours. How long would it take Paul and Matt to install the floor working together?
4 hours
9.9 hours
13.2 hours
30 hours

Answers

Answer:

9.9 hours

Step-by-step explanation:

The formula to determine the time together is

1/a+1/b = 1/c  where a and b are the times alone and c is the time together

1/18 + 1/22 = 1/c

The least common multiply of the denominators is 198c

198c(1/18 + 1/22 = 1/c)

11c+ 9c = 198

20c = 198

Divide by 20

20c/20 =198/20

c =9.9

Answer:

B - 9.9 hrs

Step-by-step explanation:

took the test.

Solve for X and show your work and explain please

Answers

Answer: x = 45

Step-by-step explanation:

Given

(2/3)x + 4 = (4/5)x - 2

Add 2 on both sides

(2/3)x + 4 + 2 = (4/5)x - 2 + 2

(2/3)x + 6 = (4/5)x

Subtract (2/3)x on both sides

(2/3)x + 6 - (2/3)x = (4/5)x - (2/3)x

6 = (12/15)x - (10/15)x

6 = (2/15)x

Divide 2/15 on both sides

6 / (2/15) = (2/15)x / (2/15)

[tex]\boxed{x=45}[/tex]

Hope this helps!! :)

Please let me know if you have any questions

Answer:

x = 45

Step-by-step explanation:

2/3 x + 4 = 4/5x - 2             Add 2 to both sides

2/3 x + 4 + 2 = 4/5x            Combine

2/3x + 6 = 4/5x                   Subtract 2/3 x from both sides.

6 = 4/5x - 2/3 x                  Multiply both sides by 15

6*15 = 4/5 x * 15 - 2/3x * 15

6*15 = 12x - 10x                   Combine the left and right

90 = 2x                               Divide by 2

x = 45

Let's see if it works.

LHS = 2/3 * 45 + 4

LHS = 2*15 + 4

LHS = 30 + 4

LHS = 34

RHS

Right hand side = 4/5 * 45 - 2

RHS = 36 - 2

RHS = 34 which is the same as the LHS

8.9 x 10^3 in standard notation

Answers

Answer:

that is n standard notation mah frand

8.9 × 10^3 being scientific notation of " 8900 "

[tex]\huge\text{Hey there!}[/tex]

[tex]\large\textsf{8.9}\times\large\textsf{10}^\mathsf{3}\\\\\mathsf{10^3}\\\mathsf{= 10\times10\times10}\\\mathsf{= 100\times10}\\\mathsf{= \bf 1,000}\\\\\large\textsf{8.9}\times\large\textsf{1,000}\\\\\large\textsf{= \bf 8,900}\\\\\\\boxed{\boxed{\huge\text{Answer: \boxed{\underline{\underline{\bf 8,900}}}}}}\huge\checkmark[/tex]

[tex]\huge\text{Good luck on your assignment \& enjoy your day!}[/tex]

~[tex]\boxed{\huge\text{}\boxed{\frak{Amphitrite1040:)}}}[/tex]

find the missing side of the triangle

Answers

Answer:

x = 34

Step-by-step explanation:

Pytago:

x[tex]30^{2} + 16^{2} = x^2\\x = \sqrt{30^2 + 16^2} \\x = 34[/tex]

Find an equation of the plane orthogonal to the line
(x,y,z)=(0,9,6)+t(7,−7,−6)

which passes through the point (9, 6, 0).

Give your answer in the form ax+by+cz=d (with a=7).

Answers

The given line is orthogonal to the plane you want to find, so the tangent vector of this line can be used as the normal vector for the plane.

The tangent vector for the line is

d/dt (⟨0, 9, 6⟩ + ⟨7, -7, -6⟩t ) = ⟨7, -7, -6⟩

Then the plane that passes through the origin with this as its normal vector has equation

x, y, z⟩ • ⟨7, -7, -6⟩ = 0

We want the plane to pass through the point (9, 6, 0), so we just translate every vector pointing to the plane itself by adding ⟨9, 6, 0⟩,

(⟨x, y, z⟩ - ⟨9, 6, 0⟩) • ⟨7, -7, -6⟩ = 0

Simplifying this expression and writing it standard form gives

x - 9, y - 6, z⟩ • ⟨7, -7, -6⟩ = 0

7 (x - 9) - 7 (y - 6) - 6z = 0

7x - 63 - 7y + 42 - 6z = 0

7x - 7y - 6z = 21

so that

a = 7, b = -7, c = -6, and d = 21

An equation of the plane orthogonal to the line 7x - 7y - 6z = 21.

The given line is orthogonal to the plane you want to find,

So the tangent vector of this line can be used as

The normal vector for the plane.

The tangent vector for the line is,

What is the tangent vector?

A tangent vector is a vector that is tangent to a curve or surface at a given point.

d/dt (⟨0, 9, 6⟩ + ⟨7, -7, -6⟩t ) = ⟨7, -7, -6⟩

Then the plane that passes through the origin with this as its normal vector has the equation

⟨x, y, z⟩ • ⟨7, -7, -6⟩ = 0

We want the plane to pass through the point (9, 6, 0), so we just

translate every vector pointing to the plane itself by adding ⟨9, 6, 0⟩,

(⟨x, y, z⟩ - ⟨9, 6, 0⟩) • ⟨7, -7, -6⟩ = 0

Simplifying this expression and writing it in standard form gives

⟨x - 9, y - 6, z⟩ • ⟨7, -7, -6⟩ = 0

7 (x - 9) - 7 (y - 6) - 6z = 0

7x - 63 - 7y + 42 - 6z = 0

7x - 7y - 6z = 21

So that, a = 7, b = -7, c = -6, and d = 21.

To learn more about the equation of plane visit:

https://brainly.com/question/1603217

State if the scenario involves a permutation or a combination. Then find the number of possibilities.

A team of 15 basketball players needs to choose two players to refill the water cooler.

Permutation/Combination:

Answer:

Answers

Answer:

Permutation ; 210 ways

Step-by-step explanation:

Permutation and combination methods refers to mathematical solution to finding the number of ways of making selection for a group of objects.

Usually, selection process whereby the order of selection does not matter are being treated using permutation, while those which takes the order of selection into cognizance are calculated using combination.

Here, selecting 2 players from 15 ; since order does not matter, we use permutation ;

Recall :

nPr = n! ÷ (n - r)!

Hence,

15P2 = 15! ÷ (15 - 2)!

15P2 = 15! ÷ 13!

15P2 = (15 * 14) = 210 ways

Reason Can you subtract a positive integer from a positive integer
and get a negive result? Explain your answer.

Answers

Answer:

No

Step-by-step explanation:

No matter the situation, when you multiply a negative by a negativeyou get a positive and a positive by a positive you get a positive. but if its two different like a negative and a positive then its NEGITIVE.

let's say you have 23 and you're multiplying by 2.

It's always increasing so it doesnt ever reach the negitive numbers.

Find the length of the arc.

A. 539π/12 km
B. 9π/3 km
C. 9π/2 km
D. 18π km

Answers

Answer:

b because it is I found out cus I took test

The length of the arc 9π/2 km.

The answer is option C.9π/2 km.

What is the arc of the circle?

The arc period of a circle can be calculated with the radius and relevant perspective using the arc period method.

  ⇒angle= arc/radius

     ⇒  135°=arc/6km

     ⇒ arc =135°*6km

     ⇒arc=135°*π/180° * 6km

    ⇒arc = 9π/2 km

Learn more about circle here:-https://brainly.com/question/24375372

#SPJ2

Find the slope of the line that goes through the
(2,6) and (-1, -6)

Answers

We can use the formula y2-y1/x2-x1 to get our slope. y2 and x2 are our second y and x coordinates, meanwhile y1 and x1 are our first y and x coordinates. -6-6/-1 -2 is -12/-3. -12/-3 is 4, the slope is 4.
the slope is 1/4 because you use the slope intercept formula

please solve the question ​

Answers

Answer:

[tex]g(-1) = -1[/tex]

[tex]g(0.75) = 0[/tex]

[tex]g(1)= 1[/tex]

Step-by-step explanation:

Given

See attachment

Solving (a): g(-1)

We make use of:

[tex]g(x) = -1[/tex]

Because: [tex]-1 \le x < 0[/tex] is true for x =-1

Hence:

[tex]g(-1) = -1[/tex]

Solving (b): g(0.75)

We make use of:

[tex]g(x) = 0[/tex]

Because: [tex]0 \le x < 1[/tex] is true for x =0.75

Hence:

[tex]g(0.75) = 0[/tex]

Solving (b): g(1)

We make use of:

[tex]g(x) = 1[/tex]

Because: [tex]1 \le x < 2[/tex] is true for x =1

Hence:

[tex]g(1)= 1[/tex]

Solve for x: 10/3 = x/(−5/2)

Answers

9514 1404 393

Answer:

  x = -25/3

Step-by-step explanation:

Multiply by the inverse of the coefficient of x. Reduce the fraction.

  (-5/2)(10/3) = (-5/2)(x/(-5/2))

  -50/6 = x = -25/3

Answer:

-25/3

Step-by-step explanation:

the other person is also correct. khan said so

Which ratio is equal to 27 : 81?

Answers

3:9 and if you reduce it again, 1:3

Answer:

1:3

Step-by-step explanation:

27 : 81

Divide each side by 27

27/27 : 81/27

1:3

Identify the transformed function that represents f(x) = ln x stretched vertically by a factor of 17, reflected across the x-axis, and shifted by 19 units left.
A. g(x) = −17ln (x + 19)
B. g(x) = 17ln (x − 19)
C. g(x) = 17ln (x + 19)
D. g(x) = −17ln (x − 19)

Answers

Answer:

b

Step-by-step explanation:

ANSWER. EXPLANATION. The given logarithmic function is. The transformation,. stretches the graph of y=f(x) vertically by a factor of c units ...

4 votes

ANSWER[tex]y = - 3 ln(x - 7) [/tex]EXPLANATIONThe given logarithmic function is [tex]f(x) = ln(x) [/tex]The transformation, [tex]y = - cf(x - k)[/tex]stretches

The length of a rectangle is twice its width. If the area of the rectangle is 72in², find its perimeter

Answers

Let breadth be x

Length=2x

[tex]\\ \sf\longmapsto Area=Length\times Breadth[/tex]

[tex]\\ \sf\longmapsto 72=2x(x)[/tex]

[tex]\\ \sf\longmapsto 2x^2=72[/tex]

[tex]\\ \sf\longmapsto x^2=\dfrac{72}{2}[/tex]

[tex]\\ \sf\longmapsto x^2=36[/tex]

[tex]\\ \sf\longmapsto x=\sqrt{36}[/tex]

[tex]\\ \sf\longmapsto x=6[/tex]

Length=6×2=12inBreadth=6in

[tex]\\ \sf\longmapsto Perimeter=2(L+B)[/tex]

[tex]\\ \sf\longmapsto Perimeter=2(12+6)[/tex]

[tex]\\ \sf\longmapsto Perimeter=2(18)[/tex]

[tex]\\ \sf\longmapsto Perimeter=36in[/tex]

Other Questions
A shop sells a particular of video recorder. Assuming that the weekly demand for the video recorder is a Poisson variable with the mean 3, find the probability that the shop sells. . (a) At least 3 in a week. (b) At most 7 in a week. (c) More than 20 in a month (4 weeks). what is the degree of the polynomial below Complete the following nuclear equations. I recommend filling in the atomic number below each symbol. Instead of using superscripts in our answers, put each answer in the corresponding box. A). 54Fe 4He --> 2 1n _____ Which of the following approaches for calculating the market value of a property involves estimating the dollar value associated with replacing the property new, as well as determining the loss in value due to physical, functional, and external obsolescence?a. income approach b. sales comparison approach c. cost approach d. Investment approach Solve this equation for x. Round your answer to the nearest hundredth. 1 = In(x + 7) josh is standing on the top of a building that is 425 feet tall. He throws a penny up into the air with an initial of 32 ft/sec. How long does it take for the penny to hit the ground?Y = -4.9x^2 + 32x + 425A. 10 seconds B. 6.25 C. 400 D. 0 seconds please help me on this 2. Explaining According to Ashoka, who can win "heavenlybliss" and how? The measure of angle tis 60 degrees.What is the x-coordinate of the point where theterminal side intersects the unit circle?12OOIsla Isla2DONE Question 7 of 40Which of the following best describes a natural resource? AsA. Currency used to exchange for or purchase productsB. A cultural or religious ideology used by humansC. Products created by humans for improved living conditionsD. A substance, energy source, or organism used by humansSUBMIT Other than the American Stroke Association, which other organization created these trainings?a. American CPR Organization b. American Heart Association c. American Cancer Societyd. American First Aid Association How have people been lately good? bad? something else? "Mt Everest is the glory of all Nepali". Justify the statement For the function F defined by F(x)=x22x+4, find F(|4|).A. 8B. 4C. 12D. 28 I need help please. The question is already there. Tysm. 20 points. Here is a list of fractions 18/45 14/30 10/25 8/20 16/40 one of these fractions are not equivalent to 2/5 write down this fractions What is the distance from point N to LM in the figure below? Select the best answer to describe the parabola Use absolute value to express the distance between -12 and -15 on the number lineA: |-12-(-15)|= -37B: |-12-(-15)|= -3C: |-12-(-15)|= 3D: |-12-(-15)|= 27 GuMagnesium metal is reacted with hydrochloric acid to producehydrogen gas. A sample of hydrogen gas is collected over waterin a eudiometer at 28.0C. The atmospheric pressure is 636mmHg. Determine the pressure (in atm) of the hydrogen gasproducedPressure =atm